Finding the Sum of a Power Series: Tips and Tricks for Success

Click For Summary
The discussion focuses on finding the sum of a power series involving the term n^2. The original poster has attempted various methods but struggles with the n^2 factor and seeks advice on whether to break it down into two series. Participants suggest using derivatives of geometric series to manipulate the expression, specifically noting that differentiating the series twice can yield the desired form. A calculation involving x=1/10 is mentioned, but there is confusion regarding the correctness of the resulting sum. The thread emphasizes the importance of careful differentiation and summation of geometric series to achieve the correct result.
peripatein
Messages
868
Reaction score
0

Homework Statement


I am trying to find the sum of the series in the attachment.


Homework Equations





The Attempt at a Solution


I have tried to use various series and their derivatives, to not much avail.
I am not sure how to handle the n^2 factor.
Should I break it down to two series?
Any suggestions?
 

Attachments

  • Sum.jpg
    Sum.jpg
    2.6 KB · Views: 485
Physics news on Phys.org
You know about derivatives? Good do this

\sum_{k=1}^\infty k^2 x^{n-1}=\left( x \left( x \sum_{k=1}^\infty x^{k-1}\right)^\prime \right)^\prime=\left( x \left( x \frac{1}{1-x}\right)^\prime \right)^\prime

|x|<1
your case will be x=1/10
 
If I am not mistaken, this yields 700/729, which, according to Wolfram, is incorrect. Would you please account for that?
 
peripatein said:
If I am not mistaken, this yields 700/729, which, according to Wolfram, is incorrect. Would you please account for that?

You are mistaken. Check it again.
 
Would you please explain how it was arrived at?
 
peripatein said:
Would you please explain how it was arrived at?

Basically you take x^(k-1). Multiplying by x and differentiating gives you k*x^(k-1). Doing the same thing again gives k^2*x^(k-1). Which is the form you want. Now sum the initial x^(k-1) as a geometric series and repeat the same sequence of operations on the function of you get.
 
Question: A clock's minute hand has length 4 and its hour hand has length 3. What is the distance between the tips at the moment when it is increasing most rapidly?(Putnam Exam Question) Answer: Making assumption that both the hands moves at constant angular velocities, the answer is ## \sqrt{7} .## But don't you think this assumption is somewhat doubtful and wrong?

Similar threads

  • · Replies 38 ·
2
Replies
38
Views
3K
  • · Replies 22 ·
Replies
22
Views
4K
Replies
4
Views
2K
Replies
7
Views
2K
Replies
8
Views
2K
  • · Replies 1 ·
Replies
1
Views
2K
  • · Replies 9 ·
Replies
9
Views
3K
  • · Replies 6 ·
Replies
6
Views
2K
Replies
29
Views
2K
  • · Replies 4 ·
Replies
4
Views
1K